- Wed Jul 13, 2016 3:03 pm
#27119
Complete Question Explanation
(The complete setup for this game can be found here: lsat/viewtopic.php?t=11115)
The correct answer choice is (A)
The question stem specifies that H is not offered. As the fourth rule lists H as a necessary condition, taking the contrapositive of the fourth rule reveals that G cannot be offered. G is not one of the answer choices, however. But, G is a necessary condition of the second rule, and taking the contrapositive of that rule reveals that L cannot be offered:
Accordingly, answer choice (A) is correct.
(The complete setup for this game can be found here: lsat/viewtopic.php?t=11115)
The correct answer choice is (A)
The question stem specifies that H is not offered. As the fourth rule lists H as a necessary condition, taking the contrapositive of the fourth rule reveals that G cannot be offered. G is not one of the answer choices, however. But, G is a necessary condition of the second rule, and taking the contrapositive of that rule reveals that L cannot be offered:
Accordingly, answer choice (A) is correct.
You do not have the required permissions to view the files attached to this post.